LSAT and Law School Admissions Forum

Get expert LSAT preparation and law school admissions advice from PowerScore Test Preparation.

User avatar
 Dave Killoran
PowerScore Staff
  • PowerScore Staff
  • Posts: 5852
  • Joined: Mar 25, 2011
|
#67426
Complete Question Explanation

Weaken-CE. The correct answer choice is (B)

The stimulus here states that although there has been a decrease in the number of hours worked weekly, the share of the population reading a newspaper has dropped in the last 20 years. At the same time, the percentage that watches television has risen by about the same amount. the conclusion that then follows is a causal one:

  • Cause ..... ..... ..... ..... ..... Effect

    TV watching up ..... :arrow: ..... Newspaper reading down
As with most causal conclusions, this one is flawed as it is not know the relationship is unquestionably true. You should then look for an answer that attacks the validity of this conclusion.


Answer choice (A): This supports the argument since it reinforces the idea that TV is a main source of news (instead of newspapers).

Answer choice (B): This is the correct answer choice. This answer shows that when the cause is absent, the effect still occurs. this suggests an alternate cause exists for the situation in the stimulus, which weakens the argument.

Answer choice (C): This answer just addresses the people who are still reading (which is not our concern), and does not address why the people are reading less, or whether reading and TV are related.

Answer choice (D): This is unfortunate, but has no effect on the argument as given!

Answer choice (E): This is the most attractive incorrect answer. Even if this answer is true, it does not attack the idea that "the percentage of the population that watches television daily has shown a similarly dramatic increase." Many more people could have TVs, so even though the time the TV is on has dropped slightly, that fact would not be undermined.
 LSAT2018
  • Posts: 242
  • Joined: Jan 10, 2018
|
#45416
I understand the correct answer (B) and was able to choose it through POE. But on a side note, I would like to ask about the numbers and percentages concepts included in the stimulus. The stimulus begins comparing the percentages of two groups, and then uses this to make a conclusion on the number of television viewing and newspaper reading. Isn't this wrong in itself? Would it be possible to get an explanation on the numbers and percentages concepts here?
 Adam Tyson
PowerScore Staff
  • PowerScore Staff
  • Posts: 5153
  • Joined: Apr 14, 2011
|
#62719
You're right, up to a point, LSAT2018. The argument here never does get around to saying anything about numbers, though, but only about changes in proportions. That is, the argument never gives any premises or conclusions about how many people do one thing or the other, but only that as one thing decreased dramatically, the other increased just as dramatically. That is evidence about an inverse proportion - one thing going up as the other goes down. That's also called a negative correlation.

From this correlation evidence, the author improperly draws a causal conclusion, and that's the flaw. Correlation, even negative correlation, never proves causation! To weaken the causal claim we might suggest an alternate cause for the correlation, or show that the correlation is not perfect in some way (sometimes one thing stays the same while the other changes, or sometimes they move in the same direction), or suggest that the supposed cause and effect are backwards, or raise doubts about the reliability of any data on which the author based his argument.

No need to delve into specific numbers here, since none were provided and none are really needed. It doesn't matter if newspaper readers went from 80% to 20% or from 60% to 40%. All that matters is that as newspaper reading went down, tv watching went up at about the same rate.
User avatar
 ashpine17
  • Posts: 321
  • Joined: Apr 06, 2021
|
#86935
I eliminated A for a different reason and I wanted to check to see if it was legit. Answer choice A states that there has been an increase in the percentage of people who TELL pollsters that TV has become their main source of news and people aren't the most honest with pollsters at times so I thought this choice wouldn't have an impact on the stimulus since the stimulus cites an actual increase in percentage or share of the population that watches TV, even if the actual number stayed the same...
User avatar
 Ryan Twomey
PowerScore Staff
  • PowerScore Staff
  • Posts: 141
  • Joined: Mar 04, 2021
|
#87020
Hey Asphine,

Using outside information on the LSAT is a tricky thing. You should only use outside information on the LSAT with very obvious facts. Like the earth is a planet or humans have thumbs or blue is a color.

I would not be able to infer that humans are automatically dishonest with polsters and thus unreliable. I would go with Dave's reaosning above. This answer choice simply strengthens the argument that television viewing has caused a decline in people reading newspaper.

I wish you all the luck in your studying and continue to post questions and look at responses.

Best,
Ryan

Get the most out of your LSAT Prep Plus subscription.

Analyze and track your performance with our Testing and Analytics Package.